What's wrong with the following proof that rearrangement of any convergent series (even a conditionally convergent one) converges

The name of the pictureThe name of the pictureThe name of the pictureClash Royale CLAN TAG#URR8PPP











up vote
0
down vote

favorite












$sum_n=1^infty a_n$ converges to $l$. Let $b_n$ be any rearrangement of $a_n$'s. We show that $sum_n=1^infty b_n$ also converges.



Let $s_k = sum_n=1^k a_n$, and $t_k = sum_n=1^k b_n$ be partial sums. Since for any $epsilon > 0$, there exists a number $N$ such that $forall n geq N$, $|s_n - l| < epsilon$. Or, equivalently $|s_m - s_N| < epsilon$ for any $m>N$. (also called Cauchy criterion)



Now, in the rearrangement series, choose an $M$ such that $a_1, a_2, dots, a_N$ are covered in $b_1, b_2, dots, b_M$. Therefore $|t_M - s_N| = |a_i_1 + a_i_2 + dots + a_i_M-N| leq |s_M' - s_N|$ for some $M' > N$. But since $|s_m - s_N| < epsilon$ for any $m>N$, we have $|t_M - s_N| < epsilon$. Consequently, $|t_M - l| < 2epsilon$. And the convergence of rearrangement follows.







share|cite|improve this question















  • 3




    You cannot conclude $ldots le |s_M'-s_N|$ because some of the non-occurring terms might have opposite sign to the occurring terms
    – Hagen von Eitzen
    Jul 22 at 17:05










  • Gotcha! Thanks.
    – student
    Jul 22 at 18:51














up vote
0
down vote

favorite












$sum_n=1^infty a_n$ converges to $l$. Let $b_n$ be any rearrangement of $a_n$'s. We show that $sum_n=1^infty b_n$ also converges.



Let $s_k = sum_n=1^k a_n$, and $t_k = sum_n=1^k b_n$ be partial sums. Since for any $epsilon > 0$, there exists a number $N$ such that $forall n geq N$, $|s_n - l| < epsilon$. Or, equivalently $|s_m - s_N| < epsilon$ for any $m>N$. (also called Cauchy criterion)



Now, in the rearrangement series, choose an $M$ such that $a_1, a_2, dots, a_N$ are covered in $b_1, b_2, dots, b_M$. Therefore $|t_M - s_N| = |a_i_1 + a_i_2 + dots + a_i_M-N| leq |s_M' - s_N|$ for some $M' > N$. But since $|s_m - s_N| < epsilon$ for any $m>N$, we have $|t_M - s_N| < epsilon$. Consequently, $|t_M - l| < 2epsilon$. And the convergence of rearrangement follows.







share|cite|improve this question















  • 3




    You cannot conclude $ldots le |s_M'-s_N|$ because some of the non-occurring terms might have opposite sign to the occurring terms
    – Hagen von Eitzen
    Jul 22 at 17:05










  • Gotcha! Thanks.
    – student
    Jul 22 at 18:51












up vote
0
down vote

favorite









up vote
0
down vote

favorite











$sum_n=1^infty a_n$ converges to $l$. Let $b_n$ be any rearrangement of $a_n$'s. We show that $sum_n=1^infty b_n$ also converges.



Let $s_k = sum_n=1^k a_n$, and $t_k = sum_n=1^k b_n$ be partial sums. Since for any $epsilon > 0$, there exists a number $N$ such that $forall n geq N$, $|s_n - l| < epsilon$. Or, equivalently $|s_m - s_N| < epsilon$ for any $m>N$. (also called Cauchy criterion)



Now, in the rearrangement series, choose an $M$ such that $a_1, a_2, dots, a_N$ are covered in $b_1, b_2, dots, b_M$. Therefore $|t_M - s_N| = |a_i_1 + a_i_2 + dots + a_i_M-N| leq |s_M' - s_N|$ for some $M' > N$. But since $|s_m - s_N| < epsilon$ for any $m>N$, we have $|t_M - s_N| < epsilon$. Consequently, $|t_M - l| < 2epsilon$. And the convergence of rearrangement follows.







share|cite|improve this question











$sum_n=1^infty a_n$ converges to $l$. Let $b_n$ be any rearrangement of $a_n$'s. We show that $sum_n=1^infty b_n$ also converges.



Let $s_k = sum_n=1^k a_n$, and $t_k = sum_n=1^k b_n$ be partial sums. Since for any $epsilon > 0$, there exists a number $N$ such that $forall n geq N$, $|s_n - l| < epsilon$. Or, equivalently $|s_m - s_N| < epsilon$ for any $m>N$. (also called Cauchy criterion)



Now, in the rearrangement series, choose an $M$ such that $a_1, a_2, dots, a_N$ are covered in $b_1, b_2, dots, b_M$. Therefore $|t_M - s_N| = |a_i_1 + a_i_2 + dots + a_i_M-N| leq |s_M' - s_N|$ for some $M' > N$. But since $|s_m - s_N| < epsilon$ for any $m>N$, we have $|t_M - s_N| < epsilon$. Consequently, $|t_M - l| < 2epsilon$. And the convergence of rearrangement follows.









share|cite|improve this question










share|cite|improve this question




share|cite|improve this question









asked Jul 22 at 16:58









student

62




62







  • 3




    You cannot conclude $ldots le |s_M'-s_N|$ because some of the non-occurring terms might have opposite sign to the occurring terms
    – Hagen von Eitzen
    Jul 22 at 17:05










  • Gotcha! Thanks.
    – student
    Jul 22 at 18:51












  • 3




    You cannot conclude $ldots le |s_M'-s_N|$ because some of the non-occurring terms might have opposite sign to the occurring terms
    – Hagen von Eitzen
    Jul 22 at 17:05










  • Gotcha! Thanks.
    – student
    Jul 22 at 18:51







3




3




You cannot conclude $ldots le |s_M'-s_N|$ because some of the non-occurring terms might have opposite sign to the occurring terms
– Hagen von Eitzen
Jul 22 at 17:05




You cannot conclude $ldots le |s_M'-s_N|$ because some of the non-occurring terms might have opposite sign to the occurring terms
– Hagen von Eitzen
Jul 22 at 17:05












Gotcha! Thanks.
– student
Jul 22 at 18:51




Gotcha! Thanks.
– student
Jul 22 at 18:51










2 Answers
2






active

oldest

votes

















up vote
2
down vote













As Hagen von Eitzen also pointed out, the problem lies in taking the following inequality:



$$
|a_i_1 + a_i_2 + ... + a_i_M-N| leq |s_M' - s_N| =
|a_N+1 + a_N+2 + ... + a_M'|
$$



The indices $i_1, ..., i_M-N$ are between $N+1$ and $M'$, and so the inequality would hold if all the $a_k$ were positive. However, this is not the case, and so we cannot take the inequality. E.g., we have:



$$
|1+1| nleq |1+(-1)+1|
$$






share|cite|improve this answer





















  • Thanks @Sambo .
    – student
    Jul 22 at 18:51

















up vote
2
down vote













Hard to say exactly what's wrong because it's hard to know exactly what you had in mind - in particular it seems to me that the $|a_i_1+dots|$ should be $|b_i_1+dots|$. But if I look at what you wrote and try to make a correct proof out of it, here's a hint regarding where the error is: If $a_1=1$, $a_2=-1$ and $a_3=1$ then $$|a_1+a_3|>|a_1+a_2+a_3|.$$






share|cite|improve this answer





















    Your Answer




    StackExchange.ifUsing("editor", function ()
    return StackExchange.using("mathjaxEditing", function ()
    StackExchange.MarkdownEditor.creationCallbacks.add(function (editor, postfix)
    StackExchange.mathjaxEditing.prepareWmdForMathJax(editor, postfix, [["$", "$"], ["\\(","\\)"]]);
    );
    );
    , "mathjax-editing");

    StackExchange.ready(function()
    var channelOptions =
    tags: "".split(" "),
    id: "69"
    ;
    initTagRenderer("".split(" "), "".split(" "), channelOptions);

    StackExchange.using("externalEditor", function()
    // Have to fire editor after snippets, if snippets enabled
    if (StackExchange.settings.snippets.snippetsEnabled)
    StackExchange.using("snippets", function()
    createEditor();
    );

    else
    createEditor();

    );

    function createEditor()
    StackExchange.prepareEditor(
    heartbeatType: 'answer',
    convertImagesToLinks: true,
    noModals: false,
    showLowRepImageUploadWarning: true,
    reputationToPostImages: 10,
    bindNavPrevention: true,
    postfix: "",
    noCode: true, onDemand: true,
    discardSelector: ".discard-answer"
    ,immediatelyShowMarkdownHelp:true
    );



    );








     

    draft saved


    draft discarded


















    StackExchange.ready(
    function ()
    StackExchange.openid.initPostLogin('.new-post-login', 'https%3a%2f%2fmath.stackexchange.com%2fquestions%2f2859568%2fwhats-wrong-with-the-following-proof-that-rearrangement-of-any-convergent-serie%23new-answer', 'question_page');

    );

    Post as a guest






























    2 Answers
    2






    active

    oldest

    votes








    2 Answers
    2






    active

    oldest

    votes









    active

    oldest

    votes






    active

    oldest

    votes








    up vote
    2
    down vote













    As Hagen von Eitzen also pointed out, the problem lies in taking the following inequality:



    $$
    |a_i_1 + a_i_2 + ... + a_i_M-N| leq |s_M' - s_N| =
    |a_N+1 + a_N+2 + ... + a_M'|
    $$



    The indices $i_1, ..., i_M-N$ are between $N+1$ and $M'$, and so the inequality would hold if all the $a_k$ were positive. However, this is not the case, and so we cannot take the inequality. E.g., we have:



    $$
    |1+1| nleq |1+(-1)+1|
    $$






    share|cite|improve this answer





















    • Thanks @Sambo .
      – student
      Jul 22 at 18:51














    up vote
    2
    down vote













    As Hagen von Eitzen also pointed out, the problem lies in taking the following inequality:



    $$
    |a_i_1 + a_i_2 + ... + a_i_M-N| leq |s_M' - s_N| =
    |a_N+1 + a_N+2 + ... + a_M'|
    $$



    The indices $i_1, ..., i_M-N$ are between $N+1$ and $M'$, and so the inequality would hold if all the $a_k$ were positive. However, this is not the case, and so we cannot take the inequality. E.g., we have:



    $$
    |1+1| nleq |1+(-1)+1|
    $$






    share|cite|improve this answer





















    • Thanks @Sambo .
      – student
      Jul 22 at 18:51












    up vote
    2
    down vote










    up vote
    2
    down vote









    As Hagen von Eitzen also pointed out, the problem lies in taking the following inequality:



    $$
    |a_i_1 + a_i_2 + ... + a_i_M-N| leq |s_M' - s_N| =
    |a_N+1 + a_N+2 + ... + a_M'|
    $$



    The indices $i_1, ..., i_M-N$ are between $N+1$ and $M'$, and so the inequality would hold if all the $a_k$ were positive. However, this is not the case, and so we cannot take the inequality. E.g., we have:



    $$
    |1+1| nleq |1+(-1)+1|
    $$






    share|cite|improve this answer













    As Hagen von Eitzen also pointed out, the problem lies in taking the following inequality:



    $$
    |a_i_1 + a_i_2 + ... + a_i_M-N| leq |s_M' - s_N| =
    |a_N+1 + a_N+2 + ... + a_M'|
    $$



    The indices $i_1, ..., i_M-N$ are between $N+1$ and $M'$, and so the inequality would hold if all the $a_k$ were positive. However, this is not the case, and so we cannot take the inequality. E.g., we have:



    $$
    |1+1| nleq |1+(-1)+1|
    $$







    share|cite|improve this answer













    share|cite|improve this answer



    share|cite|improve this answer











    answered Jul 22 at 17:10









    Sambo

    1,2561427




    1,2561427











    • Thanks @Sambo .
      – student
      Jul 22 at 18:51
















    • Thanks @Sambo .
      – student
      Jul 22 at 18:51















    Thanks @Sambo .
    – student
    Jul 22 at 18:51




    Thanks @Sambo .
    – student
    Jul 22 at 18:51










    up vote
    2
    down vote













    Hard to say exactly what's wrong because it's hard to know exactly what you had in mind - in particular it seems to me that the $|a_i_1+dots|$ should be $|b_i_1+dots|$. But if I look at what you wrote and try to make a correct proof out of it, here's a hint regarding where the error is: If $a_1=1$, $a_2=-1$ and $a_3=1$ then $$|a_1+a_3|>|a_1+a_2+a_3|.$$






    share|cite|improve this answer

























      up vote
      2
      down vote













      Hard to say exactly what's wrong because it's hard to know exactly what you had in mind - in particular it seems to me that the $|a_i_1+dots|$ should be $|b_i_1+dots|$. But if I look at what you wrote and try to make a correct proof out of it, here's a hint regarding where the error is: If $a_1=1$, $a_2=-1$ and $a_3=1$ then $$|a_1+a_3|>|a_1+a_2+a_3|.$$






      share|cite|improve this answer























        up vote
        2
        down vote










        up vote
        2
        down vote









        Hard to say exactly what's wrong because it's hard to know exactly what you had in mind - in particular it seems to me that the $|a_i_1+dots|$ should be $|b_i_1+dots|$. But if I look at what you wrote and try to make a correct proof out of it, here's a hint regarding where the error is: If $a_1=1$, $a_2=-1$ and $a_3=1$ then $$|a_1+a_3|>|a_1+a_2+a_3|.$$






        share|cite|improve this answer













        Hard to say exactly what's wrong because it's hard to know exactly what you had in mind - in particular it seems to me that the $|a_i_1+dots|$ should be $|b_i_1+dots|$. But if I look at what you wrote and try to make a correct proof out of it, here's a hint regarding where the error is: If $a_1=1$, $a_2=-1$ and $a_3=1$ then $$|a_1+a_3|>|a_1+a_2+a_3|.$$







        share|cite|improve this answer













        share|cite|improve this answer



        share|cite|improve this answer











        answered Jul 22 at 17:12









        David C. Ullrich

        54.1k33481




        54.1k33481






















             

            draft saved


            draft discarded


























             


            draft saved


            draft discarded














            StackExchange.ready(
            function ()
            StackExchange.openid.initPostLogin('.new-post-login', 'https%3a%2f%2fmath.stackexchange.com%2fquestions%2f2859568%2fwhats-wrong-with-the-following-proof-that-rearrangement-of-any-convergent-serie%23new-answer', 'question_page');

            );

            Post as a guest













































































            Comments

            Popular posts from this blog

            Color the edges and diagonals of a regular polygon

            Relationship between determinant of matrix and determinant of adjoint?

            What is the equation of a 3D cone with generalised tilt?